feat(analisi): aggiunge appunti del 28/03/2023

main
parent 323af18959
commit 6a316f0b21

@ -463,4 +463,41 @@
degli zeri, $\exists \xbar \in (x_1, x_2) \mid g(\xbar) = 0 \implies f(\xbar) = y$. Si conclude allora che anche
$y \in f(I)$, da cui la tesi.
\end{proof}
\begin{remark}
In realtà, la dimostrazione del teorema dei valori intermedi
si basa sul fatto che gli unici insiemi convessi di $\RR$ sono
gli intervalli (da sopra segue infatti che $f(I)$ è un intervallo).
%TODO: dimostrare.
\end{remark}
\begin{theorem} (di Weierstrass) Sia $I$ un intervallo chiuso\footnote{In realtà è sufficiente che $I$ sia chiuso, ossia che contenga i suoi punti
di accumulazione} e sia
$f : I \to \RRbar$ continua. Allora esistono $x_m$ e $x_M$ punti
di massimo e minimo assoluti.
\end{theorem}
\begin{proof}
Ci si limita a dimostrare l'esistenza del minimo, dacché l'esistenza
del massimo segue dal considerare $g = -f$. Sia $m := \inf f(I)$.
Esiste allora una successione $(y_n) \subseteq f(I)$ tale che
$y_n \tendston m$. Poiché $y_n \in f(I)$, $\exists x_n \in I \mid
y_n = f(x_n)$. Per il teorema di Bolzano-Weierstrass, $\exists \, (x_{n_k})
\subseteq I$ sottosuccessione convergente, ossia tale che
$x_{n_k} \to \xbar \in \RRbar$. In particolare vale che
$\xbar \in I$, dal momento che $I$ è un intervallo chiuso. %TODO: approfondire
Per la continuità di $f$ (in particolare in $\xbar$), allora $f(x_{n_k}) \tendston f(\xbar)$.
Essendo $f(x_{n_k})$ una sottosuccessione di $(y_n)$, che è
convergente, deve valere che $f(\xbar) = m$, ossia
$m \in f(I)$, da cui si ricava che $f(I)$ ammette un minimo,
ovvverosia la tesi.
\end{proof}
\begin{remark} (algoritmo max. e min.) Si consideri $\tilde{f} : \tilde{I} \to \RRbar$. Allora, poiché $\tilde{f}$ è continua ed è definita su
un intervallo chiuso, per Weierstrass ammette un massimo e un
minimo. Preso per esempio il minimo, esso potrebbe essere un
estremo di $\tilde{I}$, oppure è un punto derivabile (e quindi è
stazionario), oppure non è derivabile.
\end{remark}
\end{document}

Loading…
Cancel
Save